Could someone explain how to solve this for me?

Could Someone Explain How To Solve This For Me?

Answers

Answer 1

The answer it's 95o UwU


Related Questions

Which of the following points is a solution of y s-Ixl - 1?
O (-1,-3)
O (0,0)
O (1,-1)

Answers

Answer: The answer is (-1,-3) :)

Step-by-step explanation:

Mark me as brainlest please

What is the length of the radius of a circle with center (10, -14) that is tangent to the line
x=13?

Answers

Answer:

length of radius of a circle = 3

Step-by-step explanation:

a tangent line touches the circle at one point

if the tangent line is at x=13 (vertical line) and the center of the circle is

(x,y) = (10,-14) with its center x at 10; then take the difference to find the radius;

x(tangent line) - x(center of circle) = radius of circle

length of radius of a circle = 13-10 = 3

you can also graph the circle and draw the tangent line to find the radius

The length of the radius of a circle as described in the question is; 14.32

The center of the circle is at; (10, -14)

The line that is tangent to the circle is; x = 13.

The coordinate of a point on the line is therefore; (13, 0)

The length of the radius of the circle can.be obtained from Pythagoras theorem as;

(Radius)² = (13-10)² + (-14-0)²

(Radius)² = (3)² + (-14)²

(Radius)² = 9 + 196

Radius = √205

Radius = 14.32

Read more:

https://brainly.com/question/10443107

What’s the area for this shape??????

Answers

Answer:

40cm²

Step-by-step explanation:

The formula of a trapezium is: A=1/2(a+b)h

The values are: a=6, b=10 and h=5

Therefore, to calculate the area: A=1/2(6+10)5

Hence, the area of the trapezium is A=40cm²

what are the solutions of x2-4x+5=0
please help!!

Answers

Answer:

The solutions to the quadratic equation are:

[tex]x=i+2,\:x=-i+2[/tex]

Step-by-step explanation:

Given the equation

[tex]x^2-4x+5=0[/tex]

solving the equation

[tex]x^2-4x+5=0[/tex]

subtract 5 from both sides

[tex]x^2-4x+5-5=0-5[/tex]

[tex]x^2-4x=-5[/tex]

[tex]\mathrm{Add\:}\left(-2\right)^2\mathrm{\:to\:both\:sides}[/tex]

[tex]x^2-4x+\left(-2\right)^2=-5+\left(-2\right)^2[/tex]

[tex]\left(x-2\right)^2=-1[/tex]

[tex]\mathrm{For\:}f^2\left(x\right)=a\mathrm{\:the\:solutions\:are\:}f\left(x\right)=\sqrt{a},\:-\sqrt{a}[/tex]

solving

[tex]x-2=\sqrt{-1}[/tex]

As the imaginary rule is given by

[tex]\:\sqrt{-1}=i[/tex]

so

[tex]x-2=i[/tex]

[tex]x=i+2[/tex]

solving

[tex]x-2=-\sqrt{-1}[/tex]

[tex]x-2=-i[/tex]      ∵ [tex]\:\sqrt{-1}=i[/tex]

[tex]x=-i+2[/tex]

Therefore, the solutions to the quadratic equation are:

[tex]x=i+2,\:x=-i+2[/tex]

A rose bush of height `h` is planted at an initial height of 3 feet. The bush is expected to grow an average of 0.5 feet per year. How tall will the bush be 3 years after planting? Show the work that leads to your answer (either in the text box below or on the white board to the right).

Answers

Answer:

The height is 4.5ft

Step-by-step explanation:

Given

[tex]Initial\ Height = 3\ ft[/tex]

[tex]Growth = 0.5\ ft[/tex] yearly

Required

Determine the height in 3 years

First, we need to determine the growth equation.

Represent the height with h and the years with x

So:

[tex]y = Initial\ Height + Growth * x[/tex]

[tex]y = 3 + 0.5 * x[/tex]

[tex]y = 3 + 0.5x[/tex]

To solve the required, we have that:

[tex]x = 3[/tex]

[tex]y = 3 + 0.5x[/tex] becomes

[tex]y = 3 + 0.5 *3[/tex]

[tex]y = 3 + 1.5[/tex]

[tex]y = 4.5[/tex]

Hence, the height is 4.5ft

A baseball team had $250 to spend on supplies. The team spent $46.25 on a new bat. New baseballs cost $2 each. The inequality46.25+2b≤250 can be used to determine the number of new baseballs, b, that the team can purchase.

Answers

Answer: 101

[tex]\leq 46,25+2b\leq 250\\\\=> 2b\leq 250-46,25=203,75\\\\<=> b\leq 203,75/2=101,876[/tex]

but b is the number of new baseballs

=> [tex]b\leq 101[/tex]

=> the team can purchase 101 baseballs

Step-by-step explanation:

NEED HELP PLEASE!!! Lionel created a scatter plot and drew a line of best fit, as shown.
What is the equation of the line of best fit that Lionel drew?

Answers

Answer: A.

Y=-2x+16

Step-by-step explanation:

You gotta find the pretty points on the line of the linear function. Then you have to find the slope (rise over run) with those pretty points. Then you have to use a coordinate (I used 6,4) to find the y intercept. Then you solve the equation, by plugging in all the numbers :) hope this helps.

You kinda remind me of Joe...

Answers

Answer:

nice

Step-by-step explanation:

Answer:

ok?

Step-by-step explanation:

Which equation can be used to find 40 percent of 25?
25+ 1 25
401 40
100x4 400
40x4 16
40x4 180
25%4 100
404
100_4 25

Answers

Answer:

Yo tengo 10 años y eso da 1,190

Step-by-step explanation:

What is the slope of the line below?

Answers

Answer:

The answer is the green answer choice which is 5/7.

Step-by-step explanation:

pt2 please I will give you brainliest :/​

Answers

Answer:

I know that number four is A because the x repeats

Answer: The answer is that number four is A because the x repeats. (For both numbers.)

Step-by-step explanation:

What is the slope of the line that contains the points (-2, 2) and (3, 4)?
A.
O B. -
O c.
OD.

Answers

Answer:  m = 2/5

Step-by-step explanation:

The formula for the slope is m = rise over run = y^2 - y^1 / x^2 - x^2

which makes it 4-2 over 3-(-2) which equals 2/5

Youre answer is 2/5

Answer:

The slope of the line that contains the points (-2, 2) and (3, 4) is 2/5.

Step-by-step explanation:

The slope of a line is denoted by m and is calculated using the formula:

[tex]m = \frac{y_2-y_1}{x_2-x_1}[/tex]

Here

[tex](x_1,y_1)[/tex] are the coordinates of the first point on line and

[tex](x_2,y_2)[/tex] are the coordinates of the second point on line

Given two points are:

(x1,y1) = (-2, 2)

(x2,y2) =(3, 4)

Putting the values in the formula

[tex]m = \frac{4-2}{3-(-2)}\\m = \frac{2}{3+2}\\m=\frac{2}{5}[/tex]

Hence,

The slope of the line that contains the points (-2, 2) and (3, 4) is 2/5.

For the function below, (a) find the vertex; (b) find the axis of symmetry; (c) determine whether there is a maximum or a
minimum value and find that value; and (d) graph the function.
f(x) = -x2 - 6x-4

Answers

Answer:

Step-by-step explanation:

f(x) = ax² + bx + c

"x" coordinate of vertex is ( - [tex]\frac{b}{2a}[/tex] )

axis of symmetry is x = - [tex]\frac{b}{2a}[/tex]

If a > 0 the function opens upward and has minimum value.

If a < 0 the function opens downward and has maximum value.

f(x) = - x² - 6x - 4

(a). x = - [tex]\frac{-6}{-2}[/tex] = - 3 ; y = - (- 3)² - 6( - 3) - 4 = 5

Coordinates of vertex are (- 3, 5)

(b). The axis of symmetry is x = - 3

(c). The given function opens downward

y = 5 is a maximum of a given function.

Which statement describes the​ y-intercept of the graph of a proportional​ relationship?
Choose the correct answer below.
A.
The line intersects the​ y-axis of the graph above the origin.
B.
The line intersects the​ y-axis of the graph at the origin.
C.
It is greater than the​ x-intercept of the line.
D.
It is equal to the​ x-intercept of the line.

Answers

Answer:

B i worked out all of them and only b can fit

The line intersects the​ y-axis of the graph at the origin. The correct option is B.

What is a Proportional Relationship?

A proportional Relationship is when a variable is in direct variation to another variable such that on increasing one the other proportionally increases.

The equation for a proportional relationship is

y ∝  x

y =kx

where k is the proportionality constant

The equation of a straight line is y = mx +c

here c is the intercept on the y-axis

Comparing the two equations, this can be confirmed that

c = 0

Therefore, the line intersects the​ y-axis of the graph at the origin.

The correct option is B.

To know more about Proportional Relationship

https://brainly.com/question/12917806

#SPJ2

A total of 625 tickets were sold for the school play. They were either adult tickets or student tickets. There were 75 fewer student tickets sold than adult tickets. How many adult tickets were sold?

Answers

Answer:

75 less student tickets than adult tickets. The total is 625 tickets. So the student and adult tickets have to equal 625 when added.

Step-by-step explanation:

Student tickets: 275

Adult tickets: 350

350 adult tickets were sold


Jerry and Jo are making brownies for the bake sale.
Each pan of brownies will be cut into 18 rows with 12 brownies per row.
They plan to make 12 pans of brownies.
Choose Yes or No to tell if the equation shows the number of brownies they will make.
(12 x 18) ~ 12 = b
18 * (2 x 12) = b
18 x 12 = b
12 x (12 * 18) = b

Answers

Answer:

Yes

12 * (12 * 18) = b will give the total number of brownies to be made

Step-by-step explanation:

Here, we want to select which of the equations will give the number of brownies to be made

The total number of brownies is b

For each pan, we have 18 rows , with 12 per row

So the total number of brownies per pan is 12 * 18

Since they are making 12, the total number of brownies b will be;

12 * (12 * 18) = b

of the 24 students in Michelle's homeroom, of them rode the bus to school. How many
students in Michelle's homeroom rode the bus to school?

Answers

12 becaus math sucks

i will give brainlist

Answers

Answer:

oml not u making me do math

Step-by-step explanation:

not u making me do math

Carmen opened a bank account with
$450. Each week she deposits $60.
Write an equation to represent this situation.
60x+y=450
y=-60x+450
Oy=60x+450
y=450x+60

Answers

Y=60x+450 will be the answer

HELP PLEASE!!! This is a Function or Not a Function Assignment HELP I DON’T UNDERSTAND IT PLEASE

Answers

Answer:

O and N is a function

K is not a function.

Step-by-step explanation:

O and N is solved for X perfectly, but K has mixed variables so there isn't a y-intercept.

5. Choose the equation of a line that is perpendicular to the given line and that passes through the given
point.


4x - 12y = 2; (10,-1)

A.) y=3x + 29
B.) y=--x+29
3
C.) y=-3x + 29
D.) y=--x+7

Answers

Answer:

Step-by-step explanation:oh

Answer: C) -3x + 29. Explanation: First, you want to start by putting the given equation in point-slope form. Point slope form: y = mx + b. The equation: y = 1/3x - 1/6. From this, you know that the slope is 1/3. You need to find a line perpendicular to the equation, so find the opposite-reciprocal slope. Flip the slope fraction and make it negative: -3/1. Plug the point into an equation with this slope to find the y-intercept. -1 = -3(10) + b ; 29 = b. Plug the y-intercept, as well as the slope, back into the equation to get your answer: y = -3x + 29.

Need help fast please !!!!

Will mark as brainliest!!!!

Answers

Answer:

84 I think

Step-by-step explanation:

I I went down like a math rabbit hole I can't explain it

Answer:

VY = 84

Step-by-step explanation:

It says that S is the circumcenter of XYZ, this means that S is equidistant from the 3 vertices.

Hence,  XS = YS = ZS = 116

A number is divisible by 11 if the difference of the
sums of the alternate digits is either 0 or divisible
by 11.​

Answers

Answer:

★ A number is divisible by 11 if the sum of the digits in the odd places and the sum of the digits in the even places difference is a multiple of 11 or zero.

Step-by-step explanation:

Hope you have a great day ;-)

Answer:

the answer is 4 odd

Step-by-step explanation:

I think

Roberto has run the first 16 miles of a race, which means he is 72% finished. How long is the race?

Answers

Answer:

approximately 22.2 miles

Step-by-step explanation:

16mi/72% =xmi/100%

72x=1600

x=22.2222222.....

Answer: 22.2 miles

Step-by-step explanation:

brad wants to buy flowers for his friend with $39. The daisies are $1 each and the roses are $3 each. he buys 3 more daisies than roses. how much did the roses cost

Answers

Answer:9 roses

12 daisies

Step-by-step explanation:

Can somebody help me with this

Answers

the answer is 1 3/10

A 26-by-27-inch sheet of paper is to be used for a poster, with the shorter side at the bottom. Find the width of the margins if the printed area is to be 360 in2.

Answers

Answer:

Step-by-step explanation:

Let the width of the margins as x.

So, we subtract 'x' width on each side of the paper.

So, length = 26-x-x= 26-2 x

Width = 27-x-x=27-2 x

Now, it is given printed area is 360

We can set up equation using length x width = Area

(26-2 x)(27-2 x) = 360

Multiply the left side using FOIL method,

702 -52  x-54 x+4 x^2 =360

Combine like terms

702-106x+4x^2=360

Subtract both sides 360

4x^2-106x+342 =0

Let's divide both sides by 2 to simplify

2x^2 -53x+171=0

Let's use quadratic formula to solve.

I'll upload the file for this.

There are two possible values for x.

Approximately x= 22.46 , x=3.76

Here x=3.76 works b

first correct answer will get brainliest
Solve: 3(5-2 ) +I-5I = ?


7

15

4

14

Answers

Answer:

14

Step-by-step explanation:

when you multiply and do the absolute value thats what you get

A bird is flying at a constant rate.

This equation y = 8x + 15 can be used to represent this situation, where y is the bird's elevation in meters and x is the number of minutes the bird has been flying.

Which statement best describes the bird's elevation, given this equation?


The bird started at a height of 15 meters and is ascending at 8 meters per minute.

The bird started at a height of 8 meters and is ascending at 15 meters per minute.

The bird started at a height of 15 meters and is descending at 8 meters per minute.

The bird started at a height of 8 meters and is descending at 15 meters per minute.

Answers

Answer:

the answer is The bird started at a height of 15 meters and is ascending at 8 meters per minute.

Step-by-step explanation:

i know things

Answer:

the answer is A

Step-by-step explanation:

please explain this in steps please friends​

Answers

Answer:

See below

Step-by-step explanation:

The LHS and RHS are exactly the same, so the equality is true.

The associate property of addition says that

[tex]\boxed{a+(b+c)=(a+b)+c}[/tex]

In order to make the addition of fractions, the denominators should be same. So we can write three fractions as one.

[tex]\dfrac{1}{2} +\left(\dfrac{2}{3}+\dfrac{3}{4} \right )[/tex]

[tex]\dfrac{1}{2} +\dfrac{2}{3}+\dfrac{3}{4}[/tex]

[tex]\dfrac{1\cdot 6 }{2\cdot 6} +\dfrac{2 \cdot 4}{3 \cdot 4}+\dfrac{3 \cdot 3 }{4 \cdot 3}[/tex]

[tex]\dfrac{ 6 }{12} +\dfrac{8}{12}+\dfrac{9 }{12}[/tex]

[tex]\dfrac{ 23 }{12}[/tex]

Answer:

You gotta give me (us) more to work with homie.

Step-by-step explanation:

If I have more of a picture with it I could possibly help but all there is a barely readable and legible nonsense equation. Sorry :(

Other Questions
Please help me!The temperature of a mixture was 12Celsius at the start of the experiment. After four hours the temperature decreased 25. What is the current temperature of the mixture? Read the following statement: Line segment MN is congruent to line segment OP.Which of the following is an equivalent statement? aMN overbar is similar to OP overbar bMN overbar equals OP overbar cMN overbar is congruent to OP overbar dMN overbar is an element of OP overbar A rental car company is running two specials. Customers can pay $50 to rent a compact carfor the first day plus $6 for each additional day, or they can rent the same car for $20 thefirst day and $12 for every additional day beyond that. Nora notices that, given the numberof additional days she wants to rent the car for, the two specials are equivalent. How manyadditional days does Nora want?Write a system of equations, graph them, and type the solution who was the author of common sense? what did the pamphlet wish to accomplish or what was its purpose? INTRIPLEBDOG FASRENYOU!!!!. HAH AHAH AUBAAGGAB C All of the statements below are false. Change each statement to make it true. Use a pronounto replace the words in bold.1. Tim looks for golf balls in the ocean.He doesn't look for them in the ocean. OR He looks for them in a lake, Helpppppp pleaseeeeeSave me from this questionnnnn The sum of two numbers is 38. The smaller number is 22 less than the larger number. What are the numbers? Jim mowed 5/9 of a lawn. Sarah mowed 1/3 of the lawn. What fraction of the lawn remained to be mowed? what is Mexicos type of government, and the parts of its government. Factor (Hint: Put in Standard Form First)d^2+ 10d + 20 = - 4What is the factored form? anyone have mocks gcse for year11 maths edexcel i need more help i'm so bad at this subject z/4 - 1/2 = 9? pls steps and test Help asap----------------- 7) If x-2 is a factor of P(x)= x^2-5x+a then find the value of a2346please answer Which of the binomials below is a factor of this trinomial? X2 + 8x+16 A. X-4 B. X-8 C. X+4 D. x+8 What is meant by activity analysis? Give 3 criteria for determining whether an activity adds value. balance this chemical equation pleaseee PLEASE HELPWhat type of conflict is present as Bedford searches to find the sphere before sunset in The First Men in the Moon?man vs. selfman vs. manman vs. societyman vs. nature Which political party has the most executive orders in history?